LSAT and Law School Admissions Forum

Get expert LSAT preparation and law school admissions advice from PowerScore Test Preparation.

 Administrator
PowerScore Staff
  • PowerScore Staff
  • Posts: 8919
  • Joined: Feb 02, 2011
|
#43376
Please post your questions below! Thank you!
 mepstlsat24
  • Posts: 9
  • Joined: Feb 02, 2018
|
#43829
What's wrong with B? Doesn't this strengthen it by giving more data that people with the gene had the disks? Or is 2 people too small and doesnt strengthen it as much as C does?
 James Finch
PowerScore Staff
  • PowerScore Staff
  • Posts: 943
  • Joined: Sep 06, 2017
|
#43868
Hi mepst,

The issue with the stimulus in this question is that there is not enough evidence to show the causation that the conclusion states. What it shows is that when the effect (herniated disk) is present, so is the supposed cause (defective gene), albeit only at a 5% rate, so it couldn't possibly be the sole cause. We are also shown that when the effect is not present, the cause is not present either. What we really need to know, though, is how often the effect occurs when we have the cause.

Both answer choices (B) and (C) deal with this rate, but at opposite ends of the evidentiary strength spectrum. (B) puts the chance the cause has to actually show the effect at a mere 2% rate, making it a great answer to a Weaken question. However, as a Strengthen question, we want to show that the effect appears at as close to 1:1 with the cause as possible. (C) gives us that 1:1 cause-effect ratio, as everyone with the defective gene (cause) has a herniated disk (effect) in the study, effectively proving the conclusion correct.

Hope this helps!
User avatar
 00oo00oo00oo
  • Posts: 1
  • Joined: Apr 06, 2023
|
#100768
Hello,

Would answer B be correct if instead of the passage stating that five people had the defective gene, it was two people with the defective gene. And answer B instead of stating it was 2 they state it was 5. And answer C was not included, would that strengthen the argument? I bolded, italicized, and underlined what I mean.

Passage: Researchers examined 100 people suffering from herniated disks in their backs. two of them were found to have a defect in a particular gene. The researchers also examined 100 people who had no problems with the disks in their backs; none had the genetic defect. They concluded that the genetic defect increases the likelihood of herniated disks.

B) When the researches examined a group of 100 people with the defective gene, they found that 5 of them had herniated disks in their backs.

C) Not included.
 Robert Carroll
PowerScore Staff
  • PowerScore Staff
  • Posts: 1783
  • Joined: Dec 06, 2013
|
#100780
00oo00oo00oo,

I don't think that adjustment of answer choice (B) helps. Note that the 5 from the stimulus is 5 people with the defect out of 100 people with herniated disks. Your adjusted answer choice (B) is 5 people with herniated disks out of 100 people with the gene. That's just a different reference class - it's comparing apples and oranges. I think a much larger number in answer choice (B) might arguably help, but that number isn't necessarily 5. There's nothing special about that number if we consider 100 people with the gene - the number is "special" if we consider people with herniated disks, as shown in the stimulus, but that's because it's higher than the 0 with the gene among those with no back problems. In fact, the number isn't special at all, but instead the comparison - 5 of one reference class vs 0 of the opposite reference class.

Robert Carroll

Get the most out of your LSAT Prep Plus subscription.

Analyze and track your performance with our Testing and Analytics Package.